Found problems: 6530
2010 ELMO Shortlist, 1
For a positive integer $n$, let $\mu(n) = 0$ if $n$ is not squarefree and $(-1)^k$ if $n$ is a product of $k$ primes, and let $\sigma(n)$ be the sum of the divisors of $n$. Prove that for all $n$ we have
\[\left|\sum_{d|n}\frac{\mu(d)\sigma(d)}{d}\right| \geq \frac{1}{n}, \]
and determine when equality holds.
[i]Wenyu Cao.[/i]
1964 Polish MO Finals, 2
Prove that if $ a_1 < a_2 < \ldots < a_n $ and $ b_1 < b_2 < \ldots < b_n $, where $ n \geq 2 $, then $$\qquad (a_1 + a_2 + \ldots + a_n)(b_1 + b_2 + \ldots + b_n) < n(a_1b_1 + a_2b_2 + \ldots + a_nb_n).$$
2021 2nd Memorial "Aleksandar Blazhevski-Cane", 3
Given a positive integer $n \geq 3$, let $C_{n}$ be the collection of all $n$-tuples $a=(a_{1},a_{2},...,a_{n})$ of nonnegative reals $a_i$, $i=1,...,n$, such that $a_{1}+a_{2}+...+a_{n}=1$. For $k \in \left \{ 1,...,n-1 \right \}$ and $a \in C_{n}$, consider the sum set $\sigma_{k}(a) = \left \{a_{1}+...+a_{k},a_{2}+...+a_{k+1},...,a_{n-k+1}+...+a_{n} \right \}$.
Show the following.
(a) There exist $m_k=\max\{\min\sigma_k(a):a\in\mathcal{C}_n\}$ and $M_k=\min\{\max\sigma_k(a):a\in\mathcal{C}_n\}$.
(b) It holds that $\displaystyle{1\leq\sum_{k=1}^{n-1}(\frac{1}{M_k}-\frac{1}{m_k})\leq n-2}$. Moreover, on the left side, equality is attained only for finitely many values of $n$, whereas on the right side, equality holds for infinitely values of $n$.
2024 Moldova Team Selection Test, 7
Prove that $a=2$ is the greatest real number for which the inequality:
$$
\frac{x_1}{x_n+x_2}+\frac{x_2}{x_1+x_3}+\dots+\frac{x_n}{x_{n-1}+x_1} \ge a
$$
holds true for any $n \ge 4$ and any positive real numbers $x_1, x_2,\dots,x_n$.
2009 Romania Team Selection Test, 1
Let $ABCD$ be a circumscribed quadrilateral such that $AD>\max\{AB,BC,CD\}$, $M$ be the common point of $AB$ and $CD$ and $N$ be the common point of $AC$ and $BD$. Show that \[90^{\circ}<m(\angle AND)<90^{\circ}+\frac{1}{2}m(\angle AMD).\]
Fixed, thank you Luis.
2014 Iran Team Selection Test, 5
$n$ is a natural number. for every positive real numbers $x_{1},x_{2},...,x_{n+1}$ such that $x_{1}x_{2}...x_{n+1}=1$ prove that:
$\sqrt[x_{1}]{n}+...+\sqrt[x_{n+1}]{n} \geq n^{\sqrt[n]{x_{1}}}+...+n^{\sqrt[n]{x_{n+1}}}$
1988 Romania Team Selection Test, 2
Let $OABC$ be a trihedral angle such that \[ \angle BOC = \alpha, \quad \angle COA = \beta, \quad \angle AOB = \gamma , \quad \alpha + \beta + \gamma = \pi . \] For any interior point $P$ of the trihedral angle let $P_1$, $P_2$ and $P_3$ be the projections of $P$ on the three faces. Prove that $OP \geq PP_1+PP_2+PP_3$.
[i]Constantin Cocea[/i]
2020 Turkey Team Selection Test, 4
Let $Z^+$ be positive integers set. $f:\mathbb{Z^+}\to\mathbb{Z^+}$ is a function and
we show $ f \circ f \circ ...\circ f $ with $f_l$ for all $l\in \mathbb{Z^+}$ where $f$ is repeated $l$ times. Find all $f:\mathbb{Z^+}\to\mathbb{Z^+}$ functions such that $$ (n-1)^{2020}< \prod _{l=1}^{2020} {f_l}(n)< n^{2020}+n^{2019} $$
for all $n\in \mathbb{Z^+}$
1962 AMC 12/AHSME, 33
The set of $ x$-values satisfying the inequality $ 2 \leq |x\minus{}1| \leq 5$ is:
$ \textbf{(A)}\ \minus{}4 \leq x \leq \minus{}1 \text{ or } 3 \leq x \leq 6 \qquad
\textbf{(B)}\ 3 \leq x \leq 6 \text{ or } \minus{}6 \leq x \leq \minus{}3 \qquad
\textbf{(C)}\ x \leq \minus{}1 \text{ or } x \geq 3 \qquad
\textbf{(D)}\ \minus{}1 \leq x \leq 3 \qquad
\textbf{(E)}\ \minus{}4 \leq x \leq 6$
2001 Kazakhstan National Olympiad, 3
For positive numbers $ x_1, x_2, \ldots, x_n $ $ (n \geq 1) $ the following equality holds $$ \frac {1} {{1 + x_1}} + \frac {1} {{1 + x_2}} + \ldots + \frac {1} {{1 + x_n}} = 1. $$ Prove that $ x_1 \cdot x_2 \cdot \ldots \cdot x_n \geq (n-1) ^ n. $
2011 Brazil National Olympiad, 6
Let $a_{1}, a_{2}, a_{3}, ... a_{2011}$ be nonnegative reals with sum $\frac{2011}{2}$, prove :
$|\prod_{cyc} (a_{n} - a_{n+1})| = |(a_{1} - a_{2})(a_{2} - a_{3})...(a_{2011}-a_{1})| \le \frac{3 \sqrt3}{16}.$
2018 Balkan MO Shortlist, A3
Show that for every positive integer $n$ we have:
$$\sum_{k=0}^{n}\left(\frac{2n+1-k}{k+1}\right)^k=\left(\frac{2n+1}{1}\right)^0+\left(\frac{2n}{2}\right)^1+...+\left(\frac{n+1}{n+1}\right)^n\leq 2^n$$
[i]Proposed by Dorlir Ahmeti, Albania[/i]
2009 Romania National Olympiad, 2
Let be a real number $ a\in \left[ 2+\sqrt 2,4 \right] . $ Find $ \inf_{\stackrel{z\in\mathbb{C}}{|z|\le 1}} \left| z^2-az+a \right| . $
1999 Mongolian Mathematical Olympiad, Problem 2
Let $a,b,c$ be the real numbers with $a\ge\frac85b>0$ and $a\ge c>0$. Prove the inequality
$$\frac45\left(\frac1a+\frac1b\right)+\frac2c\ge\frac{27}2\cdot\frac1{a+b+c}.$$
MathLinks Contest 1st, 2
Let $m$ be the greatest number such that for any set of complex numbers having the sum of all modulus of all the elements $1$, there exists a subset having the modulus of the sum of the elements in the subset greater than $m$. Prove that $$\frac14 \le m \le \frac12.$$
(Optional Task for 3p) Find a smaller value for the RHS.
2019 Jozsef Wildt International Math Competition, W. 62
Prove that $$\int \limits_0^{\frac{\pi}{2}}(\cos x)^{1+\sqrt{2n+1}}dx\leq \frac{2^{n-1}n!\sqrt{\pi}}{\sqrt{2(2n+1)!}}$$for all $n\in \mathbb{N}^*$
1999 IMO, 2
Let $n \geq 2$ be a fixed integer. Find the least constant $C$ such the inequality
\[\sum_{i<j} x_{i}x_{j} \left(x^{2}_{i}+x^{2}_{j} \right) \leq C
\left(\sum_{i}x_{i} \right)^4\]
holds for any $x_{1}, \ldots ,x_{n} \geq 0$ (the sum on the left consists of $\binom{n}{2}$ summands). For this constant $C$, characterize the instances of equality.
2012 Today's Calculation Of Integral, 840
Let $x,\ y$ be real numbers. For a function $f(t)=x\sin t+y\cos t$, draw the domain of the points $(x,\ y)$ for which the following inequality holds.
\[\left|\int_{-\pi}^{\pi} f(t)\cos t\ dt\right|\leq \int_{-\pi}^{\pi} \{f(t)\}^2dt.\]
2006 Taiwan National Olympiad, 2
In triangle $ABC$, $D$ is the midpoint of side $AB$. $E$ and $F$ are points arbitrarily chosen on segments $AC$ and $BC$, respectively. Show that $[DEF] < [ADE] + [BDF]$.
2020 Greece Junior Math Olympiad, 1
Solve in real numbers $\frac{(x+2)^4}{x^3}-\frac{(x+2)^2}{2x}\ge - \frac{x}{16}$
2015 Grand Duchy of Lithuania, 1
Find all pairs of real numbers $(x, y)$ for which the inequality $y^2 + y + \sqrt{y - x^2 -xy} \le 3xy$ holds.
2004 APMO, 5
Prove that the inequality \[\left(a^{2}+2\right)\left(b^{2}+2\right)\left(c^{2}+2\right) \geq 9\left(ab+bc+ca\right)\] holds for all positive reals $a$, $b$, $c$.
2015 Hanoi Open Mathematics Competitions, 15
Let the numbers $a, b,c$ satisfy the relation $a^2+b^2+c^2 \le 8$.
Determine the maximum value of $M = 4(a^3 + b^3 + c^3) - (a^4 + b^4 + c^4)$
2012 Korea National Olympiad, 3
Let $ \{ a_1 , a_2 , \cdots, a_{10} \} = \{ 1, 2, \cdots , 10 \} $ . Find the maximum value of
\[ \sum_{n=1}^{10}(na_n ^2 - n^2 a_n ) \]
2010 Cuba MO, 7
Let $x, y, z$ be positive real numbers such that $xyz = 1$. Prove that:
$$\frac{x^3 + y^3}{x^2 + xy + y^2} +\frac{ y^3 + z^3}{y^2 + yz + z^2} + \frac{z^3 + x^3}{z^2 + zx + x^2}
\ge 2.$$